Ladder operators in quantum mechanics

In summary: Hamiltonian. In classical mechanics, the Hamiltonian is a function that describes the total energy of a system. In quantum mechanics, the Hamiltonian is a function that describes the total momentum of a system.In summary, the homework statement is asking for someone to find that the lowering operator can't generate a state of infinite norm.
  • #1
bigevil
79
0

Homework Statement



This is problem 2.11 from Griffith's QM textbook under the harmonic oscillator section.

Show that the lowering operator cannot generate a state of infinite norm, ie, [tex]\int | a_{-} \psi |^2 < \infty [/tex]

Homework Equations



This isn't so hard, except that I consistently get the wrong "sign". I have worked based on Griffith's a- operator

[tex]a_{-} = \frac{1}{\sqrt{2m}} (\frac{h}{i} \frac{d}{dx} - im\omega x)[/tex].

As suggested by the question, I am to find that [tex]\int (a_{-}\psi)^{"*"} (a_{-}\psi) dx = \int \psi^{"*"} (a_{+} a_{-} \psi) dx[/tex]. It's easy to get the answer after getting this intermediate step, but I get the wrong sign, ie, [tex]- \int \psi^{"*"} (a_{+} a_{-} \psi) dx[/tex] instead of with a positive sign.

[tex]\int (a_{-}\psi)^{"*"} (a_{-}\psi) dx = \int (\frac{1}{\sqrt{2m}}\frac{h}{i} \frac{\partial \psi^{"*"}}{\partial x} - im\omega x \psi^{"*"}) (a_{-} \psi) dx[/tex].

Once I multiply out the two brackets (which is ok because there is only one operator involved here right?) I get

[tex]\int \frac{h}{i} \frac{1}{\sqrt{2m}} (a_{-}\psi) \frac{\partial \psi^{"*"}}{\partial x} - I am \omega x a_{-} \psi \psi^{"*"} dx[/tex]

If I apply integration by parts to the first term, I get an expression along the lines of [tex]-\int \psi^{"*"} \frac{\partial \psi}{\partial x} dx[/tex]. This would transform the initial expression to [tex]-a_{+}[/tex] and leave a minus sign in the final expression!

I do know that due to the way Griffiths derives the operators, there is an alternative operator, ie [tex]a_{-} = - \frac{h}{i} \frac{d}{dx} + m\omega x[/tex]. This would turn out the correct answer I think.

But am I missing something here? Why wouldn't both methods check out equally as well? Surely the sign is important in these types of derivations.
 
Physics news on Phys.org
  • #2
Maybe you have a different edition than me, but in my book he states:
[tex]a_{\pm} = \frac{1}{\sqrt{2\hbar m \omega}}\left(\mp i p + m \omega x\right)[/tex]

This would lead to:
[tex]a_- = \frac{1}{\sqrt{2\hbar m \omega}}\left(+ ip + m\omega x\right)[/tex]

Furthermore, it is much easier to show this by noting that [itex]a_-[/itex] and [itex]a_+[/itex] are hermitian conjugates. So you can write:
[tex]\int \left| a_- \psi \right|^2 \, dx = \int (a_- \psi)^*(a_- \psi) \, dx = \int \psi^*(a_+a_- \psi) \, dx[/tex]
Then use equation 2.54:
[tex]a_+a_- = \frac{1}{\hbar \omega} H - \frac{1}{2}[/tex]
And you should be on your way to get the answer:
[tex]\int \left| a_- \psi \right|^2 \, dx = \frac{1}{\hbar \omega} E - \frac{1}{2} < \infty[/tex]
 
  • #3
I've got an older edition (ca 1995!) Nick. It's weird because Griffiths does a few examples (ie, expand a+ a-, etc) with the operator that I stated at the beginning.

I didn't know about the Hermitian part. But this particular question, which I don't think is in newer editions (I checked), requires me to do out the whole thing with integration by parts.
 
  • #4
Nick's method is the way that I would show this to myself, but, perhaps the exercise is to show this using the expressions for a± in terms of x and p.

You have to be very careful about which operators are acting on which functions.
You also need to be careful about the operator definitions.

bigevil said:
[tex]\int (a_{-}\psi)^{"*"} (a_{-}\psi) dx = \int (\frac{1}{\sqrt{2m}}\frac{h}{i} \frac{\partial \psi^{"*"}}{\partial x} - im\omega x \psi^{"*"}) (a_{-} \psi) dx[/tex]
You need to be careful how you are doing the complex conjugation.

bigevil said:
[tex]\int \frac{h}{i} \frac{1}{\sqrt{2m}} (a_{-}\psi) \frac{\partial \psi^{"*"}}{\partial x} - I am \omega x a_{-} \psi \psi^{"*"} dx[/tex]
You need to be careful about what the a- is operating on (i.e. keeping parenthesis would be a good idea).

bigevil said:
If I apply integration by parts to the first term, I get an expression along the lines of [tex]-\int \psi^{"*"} \frac{\partial \psi}{\partial x} dx[/tex].
Momentum (the partial derivative operator) does not commute with a-.

bigevil said:
I do know that due to the way Griffiths derives the operators, there is an alternative operator, ie [tex]a_{-} = - \frac{h}{i} \frac{d}{dx} + m\omega x[/tex].
I don't have Griffiths QM text, but I suspect that this is a typo. This gives a Hermitian operator, but a- should not be Hermitian if it is a lowering operator.
 
Last edited:
  • #5
There's not a typo in Griffiths. Remember to negate the first trm of the lowering operator that contains an i, as well as the second when you take the complex conjugate. I got hung up at first trying to go too far with the math. For the steps I finally used to get it correct see:

http://copaseticflow.blogspot.com/2011/08/its-obvious-not-knowing-when-not-to-do.html"

Hamilton
 
Last edited by a moderator:

1. What are ladder operators in quantum mechanics?

Ladder operators in quantum mechanics are mathematical operators used to describe the energy states of a quantum system. They were first introduced by Paul Dirac and are commonly used to study the energy levels of atoms, molecules, and other quantum systems.

2. How do ladder operators work?

Ladder operators work by raising or lowering the energy of a quantum system by discrete amounts. The raising operator increases the energy state of the system, while the lowering operator decreases it. These operators are closely related to the concept of angular momentum in quantum mechanics.

3. What is the significance of ladder operators in quantum mechanics?

Ladder operators are significant in quantum mechanics because they provide a way to describe the energy levels of a quantum system and how they change under certain conditions. They also help to simplify complex equations and make them more manageable to solve.

4. Can ladder operators be applied to all quantum systems?

Yes, ladder operators can be applied to all quantum systems, including atoms, molecules, and particles. However, the specific form of the operators may vary depending on the system being studied.

5. How are ladder operators related to quantum harmonic oscillators?

Ladder operators are closely related to quantum harmonic oscillators, which are systems that exhibit periodic motion and can be described using quantum mechanics. In fact, the ladder operators for a quantum harmonic oscillator are used to calculate the energy levels of the system.

Similar threads

  • Advanced Physics Homework Help
Replies
10
Views
555
  • Advanced Physics Homework Help
Replies
1
Views
975
  • Advanced Physics Homework Help
Replies
29
Views
107
Replies
27
Views
2K
  • Advanced Physics Homework Help
Replies
5
Views
979
  • Advanced Physics Homework Help
Replies
24
Views
783
  • Advanced Physics Homework Help
Replies
1
Views
2K
  • Advanced Physics Homework Help
Replies
3
Views
880
Replies
1
Views
1K
  • Advanced Physics Homework Help
Replies
8
Views
2K
Back
Top